Đến nội dung

minhducndc

minhducndc

Đăng ký: 11-06-2017
Offline Đăng nhập: 08-07-2018 - 20:50
****-

#701177 $​4\sum \sqrt{a^{3}b^{3}}\l...

Gửi bởi minhducndc trong 04-02-2018 - 18:26

$(b+a)^{3}= b^{3}+a^{3}+3ab(b+a)\geq b^{3}+a^{3}+6\sqrt{a^{3}b^{3}}$

Cần chứng minh

$4c^{3}+a^{3}+b^{3}+6\sqrt{a^{3}b^{3}}\geq 4(\sqrt{a^{3}b^{3}}+\sqrt{b^{3}c^{3}}+\sqrt{a^{3}c^{3}})$

$\Leftrightarrow 4c^{3}+(\sqrt{a^{3}}+\sqrt{b^{3}})^{2}\geq 4\sqrt{c^{3}}(\sqrt{a^{3}}+\sqrt{b^{3}})$(luôn đúng -AM-GM)




#700520 Cho a; b; c> 0; a 2 +b 2 +c 2 = 1 Cm: $\frac{1-ab}...

Gửi bởi minhducndc trong 19-01-2018 - 21:13

Ta có $ab\leq \frac{a^{2}+b^{2}}{2}\Rightarrow 1-ab\geq 1- \frac{a^{2}+b^{2}}{2}= \frac{1}{2}(a^{2}+b^{2}+2c^{2})$

Khi đó $\frac{1-ab}{(a+b)^{2}}\geq \frac{1}{2}.\frac{a^{2}+b^{2}+2c^{2}}{(a+b)^{2}}$

Mà $a^{2}+b^{2}\geq \frac{1}{2}(a+b)^{2}$

$\Rightarrow \sum \frac{1-ab}{(a+b)^{2}}\geq \frac{3}{4}+\sum \frac{c^{2}}{(a+b)^{2}}$

Lại có $\sum \frac{c^{2}}{(a+b)^{2}}\geq \frac{1}{3}(\sum \frac{c}{a+b})^{2}\geq \frac{1}{3}.\frac{9}{4}= \frac{3}{4}$

Đến đây suy ra đpcm.




#697865 $2(a^2+b^2+c^2)+abc+8 \geq 5(a+b+c)$

Gửi bởi minhducndc trong 06-12-2017 - 16:23

Câu 3. Đặt $(\frac{1}{a};\frac{1}{b};\frac{1}{c})= (x;y;z)\Rightarrow xyz=1$

Khi đó ta cần chứng minh $\sum a^{2}+3\geq 2\sum ab$

$\Leftrightarrow \sum a^{2}+2abc+1\geq 2\sum ab$(Vì $abc=1)$

Trong 3 số a;b;c luôn có 2 số cùng bé hoặc cùng lớn hơn 1,ko mất tính tổng quát ta giả sử 2 sô dó là b;c

$\Rightarrow a(b-1)(c-1)\geq 0$

$\Leftrightarrow abc+a\geq ab+ac\Leftrightarrow 2abc+2a+2bc\geq 2(ab+bc+ca)$

Mà $\left\{\begin{matrix} a^{2}+1\geq 2a\\ b^{2}+c^{2}\geq 2bc \end{matrix}\right.$

$\Rightarrow a^{2}+b^{2}+c^{2}+2abc+1\geq 2(ab+bc+ca)$ (Q.E.D)




#697374 Cho a,b,c >0 ;abc=1.Chứng minh: $\sum \frac{1}...

Gửi bởi minhducndc trong 28-11-2017 - 17:43

 Câu 7, Ta đặt  $(\frac{a}{a-1};\frac{b}{b-1};\frac{c}{c-1})= (x;y;z)$

Do $abc=1\Rightarrow \prod \frac{a}{a-1}= \prod \frac{1}{a-1}= (x-1)(y-1)(z-1)$

hay $xyz= (x-1)(y-1)(z-1)\Leftrightarrow x+y+z=xy+yz+zx+1$

Khi đó   $\sum \frac{a^{2}}{(a-1)^{2}}= x^{2}+y^{2}+z^{2}= (x+y+z)^{2}-2(xy+yz+zx)$

$= (x+y+z)^{2}-2(x+y+z-1)= (x+y+z)^{2}-2(x+y+z)+2\geq 1$

Q.E.D

            

 




#696805 Tìm GTNN của $P=\frac{x}{x^{2}+1}+...

Gửi bởi minhducndc trong 19-11-2017 - 09:53

Ap dụng bất đẳng thức Cauchy dạng Engel ta có

$\frac{x}{x^{2}+1}+\frac{y}{y^{2}+1}+\frac{z}{z^{2}+1}\geq \frac{(x+y+z)^{2}}{x^{3}+y^{3}+z^{3}+x+y+z}= \frac{1}{x^{3}+y^{3}+z^{3}+1}$

$\Rightarrow P\geq \frac{1}{x^{3}+y^{3}+z^{3}+1}+\frac{1}{9xyz}$

Có $\frac{1}{\sum x^{3}+1}+\frac{1}{30xyz}\geq \frac{4}{\sum x^{3}+30xyz+1}$

$\Rightarrow P\geq \frac{4}{\sum x^{3}+30xyz+1}+\frac{7}{90xyz}$

Mà  $x^{3}+y^{3}+z^{3}+24abc\leq (x+y+z)^{3}$$\Rightarrow a^{3}+b^{3}+c^{3}+30xyz\leq (x+y+z)^{3}+6xyz\leq 1+\frac{2}{9}= \frac{11}{9}$

Suy ra P>=39/10




#696780 $\frac{a^{3}}{(1+a)(1+b)}+\frac...

Gửi bởi minhducndc trong 18-11-2017 - 18:12

Vì BĐT thuần nhất nên chuẩn hóa: $a+b+c=3$

BĐT cần chứng minh trở thành: $\frac{a^2}{2a^2+(3-2a)^2}+\frac{b^2}{2b^2+(3-2b)^2}+\frac{c^2}{2c^2+(3-2c)^2}\leq 1$

Ta có đánh giá: $\frac{a^2}{2a^2+(3-2a)^2}\leq \frac{2a-1}{3}$$\Leftrightarrow 3(a-1)^2(4a-3)\geq 0$

Nếu $a,b,c\geq \frac{3}{4}$ thì đánh giá luôn đúng nên ta có: $\frac{a^2}{2a^2+(3-2a)^2}+\frac{b^2}{2b^2+(3-2b)^2}+\frac{c^2}{2c^2+(3-2c)^2}\leq \frac{2(a+b+c)-3}{3}=1$

Nếu $a,b,c<\frac{3}{4}$ thì: $\frac{a^2}{2a^2+(3-2a)^2}<\frac{1}{3}$

$\Rightarrow \frac{a^2}{2a^2+(3-2a)^2}+\frac{b^2}{2b^2+(3-2b)^2}+\frac{c^2}{2c^2+(3-2c)^2}<1$

Vậy BĐT được chứng minh:

Dấu '=' xảy ra khi: $a=b=c=1$

hình như bạn xét thiếu trg hợp nếu 2 số$\geq \frac{3}{4}$, 1số $< \frac{3}{4}$ thì sao




#696602 $\sum a$

Gửi bởi minhducndc trong 14-11-2017 - 19:33

Ta có $x^{4}+x+x\geq 3\sqrt[3]{x^{6}}= 3x^{2}$

Tương tự ta có $(x^{4}+y^{4}+z^{4})+2(x+y+z)\geq 3(x^{2}+y^{2}+z^{2})= (x^{2}+y^{2}+z^{2})^{2}$

$\Rightarrow 2(x+y+z)\geq 2(\sum x^{2}y^{2})$ (dpcm)




#696299 TOPIC thảo luận, trao đổi toán thi học sinh giỏi khối 10,11 .

Gửi bởi minhducndc trong 10-11-2017 - 16:57

Bài toán số 7 ( sưu tầm )

Tìm a để phương trình $\sqrt{a+\sqrt{a+sinx}}=sinx$ có nghiệm

Đặt $sinx= b$ $(b\in \begin{bmatrix} -1;1 \end{bmatrix})$phương trình trở thành

$\sqrt{a+\sqrt{a+b}}= b$(*)

Điều kiện $\left\{\begin{matrix} a+b\geq 0\\ b\geq 0 \end{matrix}\right.$

Khi đó  $(*)\Leftrightarrow a+\sqrt{a+b}= b^{2}$

           $\Leftrightarrow a+b+\sqrt{a+b}+\frac{1}{4}= b^{2}+b+\frac{1}{4}$

            $\Leftrightarrow (\sqrt{a+b}+\frac{1}{2})^{2}= (b+\frac{1}{2})^{2}$

            $\Leftrightarrow \sqrt{a+b}=b$ (vì $\sqrt{a+b};b> 0$)

            $\Leftrightarrow b^{2}-b-a= 0$ (**)

Có phương trình (*) có nghiệm $\Leftrightarrow pt(**)$ có nghiệm $\in \begin{bmatrix} 0;1 \end{bmatrix}$

Đến đây có thể vẽ đồ thị 2 hàm $(P):y=b^{2}-b;(d):y=a$




#696021 Cho $a,b,c>0$ và $a+b+c$=3. CMR $\sum...

Gửi bởi minhducndc trong 03-11-2017 - 20:59

Đặt bđt trên là A

$A$ $\geq \sum \frac{a^2}{a^2+2abc}$

$A$ $\geq \frac{(a+b+c)^2}{a^2+b^2+c^2+6abc}$

$A$ $\geq \frac{3}{a^2+b^2+c^2+6abc}$

Ta có $a+b+c=3$ <=> $3\geq 3\sqrt[3]{abc}$

<=> $abc\leq 1$ <=> $6abc\leq 6$

<=> $A$ $\geq \frac{9}{a^2+b^2+c^2+6}$

$a^2+b^2+c^2\geq \frac{(a+b+c)^2}{3}=3$

<=> $A$ $\geq \frac{9}{6+3}=1$

Dấu bằng xảy ra khi $a=b=c=1$

Trình bày vội, không được rõ ràng lắm.

Đoạn cuối bạn nhầm dấu rồi

Ta cần chứng minh $(a+b+c)^{2}\geq a^{2}+b^{2}+c^{2}+6abc$

$\Leftrightarrow ab+bc+ca\geq 3abc$

Ta có $abc\leq 1\Rightarrow \sqrt[3]{a^{2}b^{2}c^{2}}\geq abc$

mặt khác $ab+bc+ca\geq 3\sqrt[3]{a^{2}b^{2}c^{2}}\geq 3abc$




#695789 Cho $1\leq a,b,c\leq 3$ và $a+b+c=6$

Gửi bởi minhducndc trong 29-10-2017 - 20:23

Bài toán đưa về : Cho $1\leq a;b;c\leq 3;a+b+c=6.$ Chứng minh cho $ab+bc+ca\geq 11$

Không mất tính tổng quát ta giả sử $1\leq a\leq b\leq c\leq 3$$\Rightarrow 1\leq a\leq 2$

Khi đó $ab+bc+ca= a(b+c)+bc= a(6-a)+bc$

Do $b\leq 3;c\leq 3$$\Rightarrow (b-3)(c-3)\geq 0\Leftrightarrow bc\geq 3(b+c)-9= 3(6-a)-9$

$\Rightarrow ab+bc+ca\geq a(6-a)+3(6-a)-9= -a^{2}+3a+9$$= 11-(a-1)(a-2)\geq 11$

(Vì$1\leq a\leq 2\Rightarrow (a-1)(a-2)\leq 0$)

$\Rightarrow dpcm$




#695584 Tìm min P= $\frac{3a}{b+c}+\frac{4b...

Gửi bởi minhducndc trong 26-10-2017 - 19:20

Ta có 

$P=\frac{3a}{b+c}+\frac{4b}{c+a}+\frac{5c}{a+b}= \frac{3(a+b+c)}{b+c}+\frac{4(a+b+c)}{a+c}+\frac{5(a+b+c)}{a+b}-12$

   $= (a+b+c)(\frac{3}{b+c}+\frac{4}{a+c}+\frac{5}{a+c})-12$

Có $\frac{3}{b+c}+\frac{4}{a+c}+\frac{5}{a+b}\geq \frac{(\sqrt{3}+2+\sqrt{5})^{2}}{2(a+b+c)}$

$\Rightarrow P\geq \frac{(2+\sqrt{3}+\sqrt{5})^{2}}{2}$-12

Dâu bằng xảy ra khi$\frac{\sqrt{3}}{b+c}= \frac{2}{a+c}= \frac{\sqrt{5}}{a+b}$




#695495 $\frac{x}{y}+\frac{y}{x...

Gửi bởi minhducndc trong 25-10-2017 - 21:04

Đặt x+y-S;xy=P ta có $S^{2}\geq 4P$

$6S^{2}+8P=5S(P+3)\Rightarrow P= \frac{6S^{2}-15S}{5S-8}$

Ta cần chứng minh 

$\frac{S^{2}-2P}{P\geq }\frac{10}{3}\Leftrightarrow \frac{S^{2}}{P}\geq \frac{16}{3}$

thế P vào ta có $\frac{S^{2}(5S-8)}{6S^{2}-15S}\geq \frac{10}{3}\Leftrightarrow 5(S-4)^{2}\geq 0 (Do S\geq \frac{16}{5})$




#695470 $\frac{a^3}{b^2+3}+\frac{b^3}...

Gửi bởi minhducndc trong 25-10-2017 - 19:37

$\sum \frac{a^{3}}{b^{2}+3}= \sum \frac{a^{3}}{b^{2}+ab+bc+ca}= \sum \frac{a^{3}}{(b+c)(b+a)}.$

Áp dụng AM-GM cho 3 số $: \frac{a^{3}}{(b+a)(b+c)}+\frac{b+a}{8}+\frac{b+c}{8} \geq \frac{3a}{4}.$

Tương tự ta có đpcm.




#695171 CMR $\sum \frac{a+1}{-a^{2}+4a+1-bc...

Gửi bởi minhducndc trong 21-10-2017 - 17:37

Có $\frac{a+1}{-a^{2}+4a+1-bc}= \frac{a^{2}+a}{-a^{3}+4a^{2}+a-1}\geq \frac{1}{3a}+\frac{1}{3}$ (Vì $\frac{1}{2}\leq a\leq 4$)

Tương tự ta có $VT\geq \sum \frac{1}{3a}+1\geq 3\sqrt[3]{\frac{1}{27abc}}+1= 2$




#695006 $\sum \frac{1}{a}\geq 4\sum a$

Gửi bởi minhducndc trong 17-10-2017 - 21:12

Ta có $ab+bc+ca+2abc=1\Rightarrow \frac{1}{a+1}+\frac{1}{b+1}+\frac{1}{c+1}= 2$ (Bạn quy đồng lên)

Do đó tồn tại x,y,z sao cho $\frac{1}{a+1}= \frac{x+y}{x+y+z};\frac{1}{b+1}= \frac{y+z}{x+y+z};\frac{1}{c+1}= \frac{x+z}{x+y+z}$

(Vì $\sum \frac{y+z}{x+y+z}= 2$ )

$\Rightarrow a=\frac{z}{x+y};b=\frac{x}{y+z};c= \frac{y}{x+z}$

Như vậy ta cần chứng minh $\frac{x+y}{z}+\frac{y+z}{x}+\frac{x+z}{y}\geq 4(\frac{z}{x+y}+\frac{x}{y+z}+\frac{y}{x+z})$

Dễ thấy $\frac{4z}{x+y}\leq \frac{z}{x}+\frac{z}{y}$

Làm tương tự với các số còn lại ta được đpcm